Đến nội dung

Nghiapnh1002

Nghiapnh1002

Đăng ký: 27-01-2017
Offline Đăng nhập: 16-06-2023 - 11:47
-----

Trong chủ đề: $\left | z+2-i \right |-\left | z-2-3i \right |=...

03-05-2019 - 14:56

Gọi $O$ là gốc tọa độ ; $M,M_1,M_2$ lần lượt là các điểm biểu diễn các số phức $z;z+2-i;z-2-3i$

$OM_1-OM_2=\left | z+2-i \right |-\left | z-2-3i \right |\leqslant \left | 4+2i \right |=2\sqrt5$

Dấu bằng xảy ra khi và chỉ khi $O$ thuộc tia đối của tia $M_2M_1$. Khi đó $O$ là điểm biểu diễn số phức $z-(2+2t)-(3+t)i$ ($t\geqslant 0$)

$\Leftrightarrow z=(2+2t)+(3+t)i\Leftrightarrow \left | z \right |=\sqrt{(2+2t)^2+(3+t)^2}\geqslant \sqrt{2^2+3^2}=\sqrt{13}$

Mình không hiểu đoạn này. Chẳng phải O là gốc toạ độ sao ? Và tại sao O lại có thể biểu diễn như thế chứa


Trong chủ đề: $(a+b+c)^{2}\geq 2(a+b+c)+ab+bc+ca$

16-10-2017 - 19:29

Cho a,b,c>0 thỏa mãn $a^{2}+b^{2}+c^{2}+abc=4$.

Chứng minh rằng $(a+b+c)^{2}\geq 2(a+b+c)+ab+bc+ca$

Từ điều kiện ta có thể viết lại:

$(\frac{a}{2})^2+(\frac{b}{2})^2+(\frac{c}{2})^2+4\frac{a}{2}\frac{b}{2}\frac{c}{2}=1$

Do đó có thể đặt:

$\frac{a}{2}=cosA;\frac{b}{2}=cosB;\frac{c}{2}=cosC$

Ta viết lại biểu thức
$(\sum cosA)^2 \geq \sum cosA+\sum cosAcosB$

Mặt khác
$\sum cosA=\frac{R+r}{R}; \sum cosAcosB=\frac{p^2+r^2-4R^2}{4R^2}$

Bằng biến đổi tương đương ta có:

$p^2 \leq 4R^2+4Rr+3r^2$

Đây chính là bất đẳng thức Gerretsen !

Bạn có thể tham khảo thêm ở đây


Trong chủ đề: $X_{n}=\frac{n}{\sqrt[n]{n...

23-09-2017 - 13:24

$\lim x_{n}=e$

Chứng minh:

Ta có:

$\lim(1+\frac{1}{n})^n=e$

Áp dụng định lí trung bình Cesaro cho dãy trên ta có:

$\lim\sqrt[n]{\prod (1+\frac{1}{n})^n }=\lim\sqrt[n]{\frac{n^n}{n!}}=\lim\frac{n}{\sqrt[n]{n!}}=e$
 


Trong chủ đề: ĐỀ THI CHỌN ĐT HSG QG VÒNG 1 TỈNH ĐẮC NÔNG NĂM 2018

08-09-2017 - 21:26

Câu 2: 

a)  Ta có: $ \Sigma \frac{sinA}{sinB+sinC} = \frac {\Sigma sinA ( sinA+sinB)(sinA+sinC)}{\prod (sinA+sinB) }$ 

Ta có: $ \Sigma sinA (sinA+sinB) = (sinA+sinB+sinC)^3+3sinAsinBsinC-2(sinA+sinB+sinC)(sinAsinB+sinBsinC+sinCsinA)$

Tiếp đó áp dụng các hệ thức sau đây

$sina+sinB+sinC= \frac{p}{R}$

$\sum sinAsinB = \frac{p^2+r^2+4Rr}{4R^2}$

$\prod sinA=\frac{pr}{2R^2} $

Và bằng biến đôi tương đương cuối cùng ta cần chứng minh : 

$1<\frac{2(p^2-Rr-r^2)}{p^2+r^2+2Rr}<2$ 

Bất đẳng thức đầu tương đương: $p^2 > 4Rr+3r^2$ 

Áp dụng bất đẳng thức quen thuộc: $p^2 \geq 16Rr-5r^2$ do đó chỉ cần chứng minh $16Rr-5r^2 > 4Rr+3r^2$  

$\Rightarrow 3R>2r$ (Đúng)

Bất đẳng thức thứ 2 tương đương:

$ 2r^2+3Rr>0$ hiển nhiên.

Vậy $[M]=1$


Trong chủ đề: $(3x^{2}-3x+ 3)^{\frac{1}{3}...

26-08-2017 - 02:08

Giai pt sau:

$(3x^{2}-3x+ 3)^{\frac{1}{3}} - (\frac{x^{3}}{3}-\frac{3}{4})^{\frac{1}{2}} = \frac{1}{2}$

$\sqrt[3]{3x^2-3x+3}-\sqrt{\frac{4x^3-9}{12}}=\frac{1}{2}$ (1)

ĐKXĐ: $x \geq \sqrt[3]{\frac{9}{4}}$

 

$(x-\sqrt[3]{3x^2-3x+3})+(\sqrt\frac{4x^3-9}{12}+\frac{1}{2}-x)=0$

$\Leftrightarrow (x^3-3x^2+3x-3)(\frac{1}{x^2+x\sqrt[3]{3x^2-3x+3}+(\sqrt[3]{3x^2-3x+3})^2}+\frac{1}{3(\sqrt{\frac{4x^3-9}{12}}+(x-\frac{1}{2}))})=0$

Từ điều kiện ta có: 

$x^3-3x^2+3x-3=0$$\Leftrightarrow (x-1)^3=2$

Vậy: $x=\sqrt[3]{2}+1$ là nghiệm của phương trình.

P/s: Cảm ơn, mình đã sửa.